Evaluating $limlimits_{x to 0} frac{(1+x)^{1/x} - e + frac{1}{2}ex}{x^2}$ without expansions in limits












2












$begingroup$



Evaluate $limlimits_{x to 0} frac{(1+x)^{1/x} - e + frac{1}{2}ex}{x^2}$




One way that I can immediately think of is expanding each of the terms and solving like,
$$(1+x)^{1/x} = e^{log_e (1+x)^{1/x}} = e^{frac{1}{x} (x-frac{x^2}{2} -frac{x^3}{3}+...)}$$
and then after complete expansion of each and every and substuting into back to limit and solving I get $frac{11e}{24}$ as an answer.



Now, this is a relatively long and complicated way to solve as you can see. I want to know if there is an easier way to solve this problem. Please help. Thank you!










share|cite|improve this question











$endgroup$

















    2












    $begingroup$



    Evaluate $limlimits_{x to 0} frac{(1+x)^{1/x} - e + frac{1}{2}ex}{x^2}$




    One way that I can immediately think of is expanding each of the terms and solving like,
    $$(1+x)^{1/x} = e^{log_e (1+x)^{1/x}} = e^{frac{1}{x} (x-frac{x^2}{2} -frac{x^3}{3}+...)}$$
    and then after complete expansion of each and every and substuting into back to limit and solving I get $frac{11e}{24}$ as an answer.



    Now, this is a relatively long and complicated way to solve as you can see. I want to know if there is an easier way to solve this problem. Please help. Thank you!










    share|cite|improve this question











    $endgroup$















      2












      2








      2


      4



      $begingroup$



      Evaluate $limlimits_{x to 0} frac{(1+x)^{1/x} - e + frac{1}{2}ex}{x^2}$




      One way that I can immediately think of is expanding each of the terms and solving like,
      $$(1+x)^{1/x} = e^{log_e (1+x)^{1/x}} = e^{frac{1}{x} (x-frac{x^2}{2} -frac{x^3}{3}+...)}$$
      and then after complete expansion of each and every and substuting into back to limit and solving I get $frac{11e}{24}$ as an answer.



      Now, this is a relatively long and complicated way to solve as you can see. I want to know if there is an easier way to solve this problem. Please help. Thank you!










      share|cite|improve this question











      $endgroup$





      Evaluate $limlimits_{x to 0} frac{(1+x)^{1/x} - e + frac{1}{2}ex}{x^2}$




      One way that I can immediately think of is expanding each of the terms and solving like,
      $$(1+x)^{1/x} = e^{log_e (1+x)^{1/x}} = e^{frac{1}{x} (x-frac{x^2}{2} -frac{x^3}{3}+...)}$$
      and then after complete expansion of each and every and substuting into back to limit and solving I get $frac{11e}{24}$ as an answer.



      Now, this is a relatively long and complicated way to solve as you can see. I want to know if there is an easier way to solve this problem. Please help. Thank you!







      calculus limits






      share|cite|improve this question















      share|cite|improve this question













      share|cite|improve this question




      share|cite|improve this question








      edited Mar 12 at 7:39









      Asaf Karagila

      306k33438769




      306k33438769










      asked Mar 12 at 2:29









      rashrash

      526115




      526115






















          3 Answers
          3






          active

          oldest

          votes


















          4












          $begingroup$

          Write $f(x) = frac{1}{x}log(1+x)$ and $f(0) = 1$. We know that $f$ so defined is analytics near $0$. Now, by the L'Hospital's rule applied twice,



          begin{align*}
          lim_{xto0} frac{e^{f(x)} - e + frac{e}{2}x}{x^2}
          &= lim_{xto0} frac{e^{f(x)}f'(x) + frac{e}{2}}{2x} \
          &= lim_{xto0} frac{e^{f(x)}f''(x) + e^{f(x)}f'(x)^2}{2} \
          &= frac{e}{2}f''(0) + frac{e}{2}f'(0)^2.
          end{align*}



          Since $ f(x) = 1 - frac{1}{2}x + frac{1}{3}x^2 + cdots $ near $0$, it follows that $f'(0) = -frac{1}{2}$ and $f''(0) = frac{2}{3}$. Therefore the limit equals



          $$ frac{e}{2}cdotfrac{2}{3} + frac{e}{2}left(-frac{1}{2}right)^2
          = frac{11}{24}e. $$






          share|cite|improve this answer









          $endgroup$





















            2












            $begingroup$

            If I may suggest, the problem of
            $$y=frac{(1+x)^{frac1 x} - e + frac{1}{2}ex}{x^2}$$ is not so difficult if you use another way.
            $$a=(1+x)^{frac1 x}implies log(a)= {frac1 x}log(1+x)$$
            $$ log(a)={frac1 x}left(x-frac{x^2}{2}+frac{x^3}{3}-frac{x^4}{4}+Oleft(x^5right) right)=1-frac{x}{2}+frac{x^2}{3}-frac{x^3}{4}+Oleft(x^4right)$$ Now, continuing with Taylor
            $$a=e^{log(a)}=e-frac{e x}{2}+frac{11 e x^2}{24}-frac{7 e x^3}{16}+Oleft(x^4right)$$
            $$y=frac{frac{11 e x^2}{24}-frac{7 e x^3}{16}+Oleft(x^4right) }{x^2}=frac{11 e}{24}-frac{7 e x}{16}+Oleft(x^2right)$$ which gives not only the limit but also how it is approached.






            share|cite|improve this answer









            $endgroup$





















              1












              $begingroup$

              Solution without expansions by the L'Hospital's rule only:
              $$lim_{xrightarrow0}frac{(1+x)^{1/x} - e + frac{1}{2}ex}{x^2}=lim_{xrightarrow0}frac{(1+x)^{frac{1}{x}}left(frac{ln(1+x)}{x}right)'+frac{1}{2}e}{2x}=$$
              $$=lim_{xrightarrow0}frac{frac{(1+x)^{frac{1}{x}}left(frac{x}{1+x}-ln(1+x)right)}{x^2}+frac{1}{2}e}{2x}=lim_{xrightarrow0}frac{left(frac{(1+x)^{frac{1}{x}}left(frac{x}{1+x}-ln(1+x)right)}{x^2}right)'}{2}$$ because
              $$lim_{xrightarrow0}frac{(1+x)^{frac{1}{x}}left(frac{x}{1+x}-ln(1+x)right)}{x^2}=elim_{xrightarrow0}frac{x-(1+x)ln(1+x)}{x^2+x^3}=$$
              $$=elim_{xrightarrow0}frac{1-ln(1+x)-1}{2x+3x^2}=-frac{e}{2}$$ and we can continue:
              $$ lim_{xrightarrow0}frac{left(frac{(1+x)^{frac{1}{x}}left(frac{x}{1+x}-ln(1+x)right)}{x^2}right)'}{2}=$$
              $$=frac{1}{2}lim_{xrightarrow0}left(frac{frac{(1+x)^{frac{1}{x}}left(frac{x}{1+x}-ln(1+x)right)^2}{x^2}+(1+x)^{frac{1}{x}}left(frac{1}{(1+x)^2}-frac{1}{1+x}right)}{x^2}-frac{2}{x^3}(1+x)^{frac{1}{x}}left(frac{x}{1+x}-ln(1+x)right)right)=$$
              $$=frac{e}{2}lim_{xrightarrow}left(frac{frac{left(frac{x}{1+x}-ln(1+x)right)^2}{x^2}-frac{x}{(1+x)^2}}{x^2}-frac{2}{x^3}left(frac{x}{1+x}-ln(1+x)right)right)=$$
              $$=frac{e}{2}lim_{xrightarrow0}left(-frac{3x+1}{x^2(1+x)^2}+frac{2ln(1+x)}{(1+x)x^2}+frac{ln^2(1+x)}{x^4}right)=$$
              $$=frac{e}{2}lim_{xrightarrow0}frac{(1+x)^2ln^2(1+x)+(2x^3+2x^2)ln(1+x)-3x^3-x^2}{x^4}=...=frac{11e}{24}.$$






              share|cite|improve this answer









              $endgroup$













                Your Answer





                StackExchange.ifUsing("editor", function () {
                return StackExchange.using("mathjaxEditing", function () {
                StackExchange.MarkdownEditor.creationCallbacks.add(function (editor, postfix) {
                StackExchange.mathjaxEditing.prepareWmdForMathJax(editor, postfix, [["$", "$"], ["\\(","\\)"]]);
                });
                });
                }, "mathjax-editing");

                StackExchange.ready(function() {
                var channelOptions = {
                tags: "".split(" "),
                id: "69"
                };
                initTagRenderer("".split(" "), "".split(" "), channelOptions);

                StackExchange.using("externalEditor", function() {
                // Have to fire editor after snippets, if snippets enabled
                if (StackExchange.settings.snippets.snippetsEnabled) {
                StackExchange.using("snippets", function() {
                createEditor();
                });
                }
                else {
                createEditor();
                }
                });

                function createEditor() {
                StackExchange.prepareEditor({
                heartbeatType: 'answer',
                autoActivateHeartbeat: false,
                convertImagesToLinks: true,
                noModals: true,
                showLowRepImageUploadWarning: true,
                reputationToPostImages: 10,
                bindNavPrevention: true,
                postfix: "",
                imageUploader: {
                brandingHtml: "Powered by u003ca class="icon-imgur-white" href="https://imgur.com/"u003eu003c/au003e",
                contentPolicyHtml: "User contributions licensed under u003ca href="https://creativecommons.org/licenses/by-sa/3.0/"u003ecc by-sa 3.0 with attribution requiredu003c/au003e u003ca href="https://stackoverflow.com/legal/content-policy"u003e(content policy)u003c/au003e",
                allowUrls: true
                },
                noCode: true, onDemand: true,
                discardSelector: ".discard-answer"
                ,immediatelyShowMarkdownHelp:true
                });


                }
                });














                draft saved

                draft discarded


















                StackExchange.ready(
                function () {
                StackExchange.openid.initPostLogin('.new-post-login', 'https%3a%2f%2fmath.stackexchange.com%2fquestions%2f3144560%2fevaluating-lim-limits-x-to-0-frac1x1-x-e-frac12exx2-w%23new-answer', 'question_page');
                }
                );

                Post as a guest















                Required, but never shown

























                3 Answers
                3






                active

                oldest

                votes








                3 Answers
                3






                active

                oldest

                votes









                active

                oldest

                votes






                active

                oldest

                votes









                4












                $begingroup$

                Write $f(x) = frac{1}{x}log(1+x)$ and $f(0) = 1$. We know that $f$ so defined is analytics near $0$. Now, by the L'Hospital's rule applied twice,



                begin{align*}
                lim_{xto0} frac{e^{f(x)} - e + frac{e}{2}x}{x^2}
                &= lim_{xto0} frac{e^{f(x)}f'(x) + frac{e}{2}}{2x} \
                &= lim_{xto0} frac{e^{f(x)}f''(x) + e^{f(x)}f'(x)^2}{2} \
                &= frac{e}{2}f''(0) + frac{e}{2}f'(0)^2.
                end{align*}



                Since $ f(x) = 1 - frac{1}{2}x + frac{1}{3}x^2 + cdots $ near $0$, it follows that $f'(0) = -frac{1}{2}$ and $f''(0) = frac{2}{3}$. Therefore the limit equals



                $$ frac{e}{2}cdotfrac{2}{3} + frac{e}{2}left(-frac{1}{2}right)^2
                = frac{11}{24}e. $$






                share|cite|improve this answer









                $endgroup$


















                  4












                  $begingroup$

                  Write $f(x) = frac{1}{x}log(1+x)$ and $f(0) = 1$. We know that $f$ so defined is analytics near $0$. Now, by the L'Hospital's rule applied twice,



                  begin{align*}
                  lim_{xto0} frac{e^{f(x)} - e + frac{e}{2}x}{x^2}
                  &= lim_{xto0} frac{e^{f(x)}f'(x) + frac{e}{2}}{2x} \
                  &= lim_{xto0} frac{e^{f(x)}f''(x) + e^{f(x)}f'(x)^2}{2} \
                  &= frac{e}{2}f''(0) + frac{e}{2}f'(0)^2.
                  end{align*}



                  Since $ f(x) = 1 - frac{1}{2}x + frac{1}{3}x^2 + cdots $ near $0$, it follows that $f'(0) = -frac{1}{2}$ and $f''(0) = frac{2}{3}$. Therefore the limit equals



                  $$ frac{e}{2}cdotfrac{2}{3} + frac{e}{2}left(-frac{1}{2}right)^2
                  = frac{11}{24}e. $$






                  share|cite|improve this answer









                  $endgroup$
















                    4












                    4








                    4





                    $begingroup$

                    Write $f(x) = frac{1}{x}log(1+x)$ and $f(0) = 1$. We know that $f$ so defined is analytics near $0$. Now, by the L'Hospital's rule applied twice,



                    begin{align*}
                    lim_{xto0} frac{e^{f(x)} - e + frac{e}{2}x}{x^2}
                    &= lim_{xto0} frac{e^{f(x)}f'(x) + frac{e}{2}}{2x} \
                    &= lim_{xto0} frac{e^{f(x)}f''(x) + e^{f(x)}f'(x)^2}{2} \
                    &= frac{e}{2}f''(0) + frac{e}{2}f'(0)^2.
                    end{align*}



                    Since $ f(x) = 1 - frac{1}{2}x + frac{1}{3}x^2 + cdots $ near $0$, it follows that $f'(0) = -frac{1}{2}$ and $f''(0) = frac{2}{3}$. Therefore the limit equals



                    $$ frac{e}{2}cdotfrac{2}{3} + frac{e}{2}left(-frac{1}{2}right)^2
                    = frac{11}{24}e. $$






                    share|cite|improve this answer









                    $endgroup$



                    Write $f(x) = frac{1}{x}log(1+x)$ and $f(0) = 1$. We know that $f$ so defined is analytics near $0$. Now, by the L'Hospital's rule applied twice,



                    begin{align*}
                    lim_{xto0} frac{e^{f(x)} - e + frac{e}{2}x}{x^2}
                    &= lim_{xto0} frac{e^{f(x)}f'(x) + frac{e}{2}}{2x} \
                    &= lim_{xto0} frac{e^{f(x)}f''(x) + e^{f(x)}f'(x)^2}{2} \
                    &= frac{e}{2}f''(0) + frac{e}{2}f'(0)^2.
                    end{align*}



                    Since $ f(x) = 1 - frac{1}{2}x + frac{1}{3}x^2 + cdots $ near $0$, it follows that $f'(0) = -frac{1}{2}$ and $f''(0) = frac{2}{3}$. Therefore the limit equals



                    $$ frac{e}{2}cdotfrac{2}{3} + frac{e}{2}left(-frac{1}{2}right)^2
                    = frac{11}{24}e. $$







                    share|cite|improve this answer












                    share|cite|improve this answer



                    share|cite|improve this answer










                    answered Mar 12 at 6:39









                    Sangchul LeeSangchul Lee

                    96.1k12171281




                    96.1k12171281























                        2












                        $begingroup$

                        If I may suggest, the problem of
                        $$y=frac{(1+x)^{frac1 x} - e + frac{1}{2}ex}{x^2}$$ is not so difficult if you use another way.
                        $$a=(1+x)^{frac1 x}implies log(a)= {frac1 x}log(1+x)$$
                        $$ log(a)={frac1 x}left(x-frac{x^2}{2}+frac{x^3}{3}-frac{x^4}{4}+Oleft(x^5right) right)=1-frac{x}{2}+frac{x^2}{3}-frac{x^3}{4}+Oleft(x^4right)$$ Now, continuing with Taylor
                        $$a=e^{log(a)}=e-frac{e x}{2}+frac{11 e x^2}{24}-frac{7 e x^3}{16}+Oleft(x^4right)$$
                        $$y=frac{frac{11 e x^2}{24}-frac{7 e x^3}{16}+Oleft(x^4right) }{x^2}=frac{11 e}{24}-frac{7 e x}{16}+Oleft(x^2right)$$ which gives not only the limit but also how it is approached.






                        share|cite|improve this answer









                        $endgroup$


















                          2












                          $begingroup$

                          If I may suggest, the problem of
                          $$y=frac{(1+x)^{frac1 x} - e + frac{1}{2}ex}{x^2}$$ is not so difficult if you use another way.
                          $$a=(1+x)^{frac1 x}implies log(a)= {frac1 x}log(1+x)$$
                          $$ log(a)={frac1 x}left(x-frac{x^2}{2}+frac{x^3}{3}-frac{x^4}{4}+Oleft(x^5right) right)=1-frac{x}{2}+frac{x^2}{3}-frac{x^3}{4}+Oleft(x^4right)$$ Now, continuing with Taylor
                          $$a=e^{log(a)}=e-frac{e x}{2}+frac{11 e x^2}{24}-frac{7 e x^3}{16}+Oleft(x^4right)$$
                          $$y=frac{frac{11 e x^2}{24}-frac{7 e x^3}{16}+Oleft(x^4right) }{x^2}=frac{11 e}{24}-frac{7 e x}{16}+Oleft(x^2right)$$ which gives not only the limit but also how it is approached.






                          share|cite|improve this answer









                          $endgroup$
















                            2












                            2








                            2





                            $begingroup$

                            If I may suggest, the problem of
                            $$y=frac{(1+x)^{frac1 x} - e + frac{1}{2}ex}{x^2}$$ is not so difficult if you use another way.
                            $$a=(1+x)^{frac1 x}implies log(a)= {frac1 x}log(1+x)$$
                            $$ log(a)={frac1 x}left(x-frac{x^2}{2}+frac{x^3}{3}-frac{x^4}{4}+Oleft(x^5right) right)=1-frac{x}{2}+frac{x^2}{3}-frac{x^3}{4}+Oleft(x^4right)$$ Now, continuing with Taylor
                            $$a=e^{log(a)}=e-frac{e x}{2}+frac{11 e x^2}{24}-frac{7 e x^3}{16}+Oleft(x^4right)$$
                            $$y=frac{frac{11 e x^2}{24}-frac{7 e x^3}{16}+Oleft(x^4right) }{x^2}=frac{11 e}{24}-frac{7 e x}{16}+Oleft(x^2right)$$ which gives not only the limit but also how it is approached.






                            share|cite|improve this answer









                            $endgroup$



                            If I may suggest, the problem of
                            $$y=frac{(1+x)^{frac1 x} - e + frac{1}{2}ex}{x^2}$$ is not so difficult if you use another way.
                            $$a=(1+x)^{frac1 x}implies log(a)= {frac1 x}log(1+x)$$
                            $$ log(a)={frac1 x}left(x-frac{x^2}{2}+frac{x^3}{3}-frac{x^4}{4}+Oleft(x^5right) right)=1-frac{x}{2}+frac{x^2}{3}-frac{x^3}{4}+Oleft(x^4right)$$ Now, continuing with Taylor
                            $$a=e^{log(a)}=e-frac{e x}{2}+frac{11 e x^2}{24}-frac{7 e x^3}{16}+Oleft(x^4right)$$
                            $$y=frac{frac{11 e x^2}{24}-frac{7 e x^3}{16}+Oleft(x^4right) }{x^2}=frac{11 e}{24}-frac{7 e x}{16}+Oleft(x^2right)$$ which gives not only the limit but also how it is approached.







                            share|cite|improve this answer












                            share|cite|improve this answer



                            share|cite|improve this answer










                            answered Mar 12 at 4:31









                            Claude LeiboviciClaude Leibovici

                            124k1158135




                            124k1158135























                                1












                                $begingroup$

                                Solution without expansions by the L'Hospital's rule only:
                                $$lim_{xrightarrow0}frac{(1+x)^{1/x} - e + frac{1}{2}ex}{x^2}=lim_{xrightarrow0}frac{(1+x)^{frac{1}{x}}left(frac{ln(1+x)}{x}right)'+frac{1}{2}e}{2x}=$$
                                $$=lim_{xrightarrow0}frac{frac{(1+x)^{frac{1}{x}}left(frac{x}{1+x}-ln(1+x)right)}{x^2}+frac{1}{2}e}{2x}=lim_{xrightarrow0}frac{left(frac{(1+x)^{frac{1}{x}}left(frac{x}{1+x}-ln(1+x)right)}{x^2}right)'}{2}$$ because
                                $$lim_{xrightarrow0}frac{(1+x)^{frac{1}{x}}left(frac{x}{1+x}-ln(1+x)right)}{x^2}=elim_{xrightarrow0}frac{x-(1+x)ln(1+x)}{x^2+x^3}=$$
                                $$=elim_{xrightarrow0}frac{1-ln(1+x)-1}{2x+3x^2}=-frac{e}{2}$$ and we can continue:
                                $$ lim_{xrightarrow0}frac{left(frac{(1+x)^{frac{1}{x}}left(frac{x}{1+x}-ln(1+x)right)}{x^2}right)'}{2}=$$
                                $$=frac{1}{2}lim_{xrightarrow0}left(frac{frac{(1+x)^{frac{1}{x}}left(frac{x}{1+x}-ln(1+x)right)^2}{x^2}+(1+x)^{frac{1}{x}}left(frac{1}{(1+x)^2}-frac{1}{1+x}right)}{x^2}-frac{2}{x^3}(1+x)^{frac{1}{x}}left(frac{x}{1+x}-ln(1+x)right)right)=$$
                                $$=frac{e}{2}lim_{xrightarrow}left(frac{frac{left(frac{x}{1+x}-ln(1+x)right)^2}{x^2}-frac{x}{(1+x)^2}}{x^2}-frac{2}{x^3}left(frac{x}{1+x}-ln(1+x)right)right)=$$
                                $$=frac{e}{2}lim_{xrightarrow0}left(-frac{3x+1}{x^2(1+x)^2}+frac{2ln(1+x)}{(1+x)x^2}+frac{ln^2(1+x)}{x^4}right)=$$
                                $$=frac{e}{2}lim_{xrightarrow0}frac{(1+x)^2ln^2(1+x)+(2x^3+2x^2)ln(1+x)-3x^3-x^2}{x^4}=...=frac{11e}{24}.$$






                                share|cite|improve this answer









                                $endgroup$


















                                  1












                                  $begingroup$

                                  Solution without expansions by the L'Hospital's rule only:
                                  $$lim_{xrightarrow0}frac{(1+x)^{1/x} - e + frac{1}{2}ex}{x^2}=lim_{xrightarrow0}frac{(1+x)^{frac{1}{x}}left(frac{ln(1+x)}{x}right)'+frac{1}{2}e}{2x}=$$
                                  $$=lim_{xrightarrow0}frac{frac{(1+x)^{frac{1}{x}}left(frac{x}{1+x}-ln(1+x)right)}{x^2}+frac{1}{2}e}{2x}=lim_{xrightarrow0}frac{left(frac{(1+x)^{frac{1}{x}}left(frac{x}{1+x}-ln(1+x)right)}{x^2}right)'}{2}$$ because
                                  $$lim_{xrightarrow0}frac{(1+x)^{frac{1}{x}}left(frac{x}{1+x}-ln(1+x)right)}{x^2}=elim_{xrightarrow0}frac{x-(1+x)ln(1+x)}{x^2+x^3}=$$
                                  $$=elim_{xrightarrow0}frac{1-ln(1+x)-1}{2x+3x^2}=-frac{e}{2}$$ and we can continue:
                                  $$ lim_{xrightarrow0}frac{left(frac{(1+x)^{frac{1}{x}}left(frac{x}{1+x}-ln(1+x)right)}{x^2}right)'}{2}=$$
                                  $$=frac{1}{2}lim_{xrightarrow0}left(frac{frac{(1+x)^{frac{1}{x}}left(frac{x}{1+x}-ln(1+x)right)^2}{x^2}+(1+x)^{frac{1}{x}}left(frac{1}{(1+x)^2}-frac{1}{1+x}right)}{x^2}-frac{2}{x^3}(1+x)^{frac{1}{x}}left(frac{x}{1+x}-ln(1+x)right)right)=$$
                                  $$=frac{e}{2}lim_{xrightarrow}left(frac{frac{left(frac{x}{1+x}-ln(1+x)right)^2}{x^2}-frac{x}{(1+x)^2}}{x^2}-frac{2}{x^3}left(frac{x}{1+x}-ln(1+x)right)right)=$$
                                  $$=frac{e}{2}lim_{xrightarrow0}left(-frac{3x+1}{x^2(1+x)^2}+frac{2ln(1+x)}{(1+x)x^2}+frac{ln^2(1+x)}{x^4}right)=$$
                                  $$=frac{e}{2}lim_{xrightarrow0}frac{(1+x)^2ln^2(1+x)+(2x^3+2x^2)ln(1+x)-3x^3-x^2}{x^4}=...=frac{11e}{24}.$$






                                  share|cite|improve this answer









                                  $endgroup$
















                                    1












                                    1








                                    1





                                    $begingroup$

                                    Solution without expansions by the L'Hospital's rule only:
                                    $$lim_{xrightarrow0}frac{(1+x)^{1/x} - e + frac{1}{2}ex}{x^2}=lim_{xrightarrow0}frac{(1+x)^{frac{1}{x}}left(frac{ln(1+x)}{x}right)'+frac{1}{2}e}{2x}=$$
                                    $$=lim_{xrightarrow0}frac{frac{(1+x)^{frac{1}{x}}left(frac{x}{1+x}-ln(1+x)right)}{x^2}+frac{1}{2}e}{2x}=lim_{xrightarrow0}frac{left(frac{(1+x)^{frac{1}{x}}left(frac{x}{1+x}-ln(1+x)right)}{x^2}right)'}{2}$$ because
                                    $$lim_{xrightarrow0}frac{(1+x)^{frac{1}{x}}left(frac{x}{1+x}-ln(1+x)right)}{x^2}=elim_{xrightarrow0}frac{x-(1+x)ln(1+x)}{x^2+x^3}=$$
                                    $$=elim_{xrightarrow0}frac{1-ln(1+x)-1}{2x+3x^2}=-frac{e}{2}$$ and we can continue:
                                    $$ lim_{xrightarrow0}frac{left(frac{(1+x)^{frac{1}{x}}left(frac{x}{1+x}-ln(1+x)right)}{x^2}right)'}{2}=$$
                                    $$=frac{1}{2}lim_{xrightarrow0}left(frac{frac{(1+x)^{frac{1}{x}}left(frac{x}{1+x}-ln(1+x)right)^2}{x^2}+(1+x)^{frac{1}{x}}left(frac{1}{(1+x)^2}-frac{1}{1+x}right)}{x^2}-frac{2}{x^3}(1+x)^{frac{1}{x}}left(frac{x}{1+x}-ln(1+x)right)right)=$$
                                    $$=frac{e}{2}lim_{xrightarrow}left(frac{frac{left(frac{x}{1+x}-ln(1+x)right)^2}{x^2}-frac{x}{(1+x)^2}}{x^2}-frac{2}{x^3}left(frac{x}{1+x}-ln(1+x)right)right)=$$
                                    $$=frac{e}{2}lim_{xrightarrow0}left(-frac{3x+1}{x^2(1+x)^2}+frac{2ln(1+x)}{(1+x)x^2}+frac{ln^2(1+x)}{x^4}right)=$$
                                    $$=frac{e}{2}lim_{xrightarrow0}frac{(1+x)^2ln^2(1+x)+(2x^3+2x^2)ln(1+x)-3x^3-x^2}{x^4}=...=frac{11e}{24}.$$






                                    share|cite|improve this answer









                                    $endgroup$



                                    Solution without expansions by the L'Hospital's rule only:
                                    $$lim_{xrightarrow0}frac{(1+x)^{1/x} - e + frac{1}{2}ex}{x^2}=lim_{xrightarrow0}frac{(1+x)^{frac{1}{x}}left(frac{ln(1+x)}{x}right)'+frac{1}{2}e}{2x}=$$
                                    $$=lim_{xrightarrow0}frac{frac{(1+x)^{frac{1}{x}}left(frac{x}{1+x}-ln(1+x)right)}{x^2}+frac{1}{2}e}{2x}=lim_{xrightarrow0}frac{left(frac{(1+x)^{frac{1}{x}}left(frac{x}{1+x}-ln(1+x)right)}{x^2}right)'}{2}$$ because
                                    $$lim_{xrightarrow0}frac{(1+x)^{frac{1}{x}}left(frac{x}{1+x}-ln(1+x)right)}{x^2}=elim_{xrightarrow0}frac{x-(1+x)ln(1+x)}{x^2+x^3}=$$
                                    $$=elim_{xrightarrow0}frac{1-ln(1+x)-1}{2x+3x^2}=-frac{e}{2}$$ and we can continue:
                                    $$ lim_{xrightarrow0}frac{left(frac{(1+x)^{frac{1}{x}}left(frac{x}{1+x}-ln(1+x)right)}{x^2}right)'}{2}=$$
                                    $$=frac{1}{2}lim_{xrightarrow0}left(frac{frac{(1+x)^{frac{1}{x}}left(frac{x}{1+x}-ln(1+x)right)^2}{x^2}+(1+x)^{frac{1}{x}}left(frac{1}{(1+x)^2}-frac{1}{1+x}right)}{x^2}-frac{2}{x^3}(1+x)^{frac{1}{x}}left(frac{x}{1+x}-ln(1+x)right)right)=$$
                                    $$=frac{e}{2}lim_{xrightarrow}left(frac{frac{left(frac{x}{1+x}-ln(1+x)right)^2}{x^2}-frac{x}{(1+x)^2}}{x^2}-frac{2}{x^3}left(frac{x}{1+x}-ln(1+x)right)right)=$$
                                    $$=frac{e}{2}lim_{xrightarrow0}left(-frac{3x+1}{x^2(1+x)^2}+frac{2ln(1+x)}{(1+x)x^2}+frac{ln^2(1+x)}{x^4}right)=$$
                                    $$=frac{e}{2}lim_{xrightarrow0}frac{(1+x)^2ln^2(1+x)+(2x^3+2x^2)ln(1+x)-3x^3-x^2}{x^4}=...=frac{11e}{24}.$$







                                    share|cite|improve this answer












                                    share|cite|improve this answer



                                    share|cite|improve this answer










                                    answered Mar 12 at 8:57









                                    Michael RozenbergMichael Rozenberg

                                    108k1895200




                                    108k1895200






























                                        draft saved

                                        draft discarded




















































                                        Thanks for contributing an answer to Mathematics Stack Exchange!


                                        • Please be sure to answer the question. Provide details and share your research!

                                        But avoid



                                        • Asking for help, clarification, or responding to other answers.

                                        • Making statements based on opinion; back them up with references or personal experience.


                                        Use MathJax to format equations. MathJax reference.


                                        To learn more, see our tips on writing great answers.




                                        draft saved


                                        draft discarded














                                        StackExchange.ready(
                                        function () {
                                        StackExchange.openid.initPostLogin('.new-post-login', 'https%3a%2f%2fmath.stackexchange.com%2fquestions%2f3144560%2fevaluating-lim-limits-x-to-0-frac1x1-x-e-frac12exx2-w%23new-answer', 'question_page');
                                        }
                                        );

                                        Post as a guest















                                        Required, but never shown





















































                                        Required, but never shown














                                        Required, but never shown












                                        Required, but never shown







                                        Required, but never shown

































                                        Required, but never shown














                                        Required, but never shown












                                        Required, but never shown







                                        Required, but never shown







                                        Popular posts from this blog

                                        Biblatex bibliography style without URLs when DOI exists (in Overleaf with Zotero bibliography)

                                        ComboBox Display Member on multiple fields

                                        Is it possible to collect Nectar points via Trainline?